CRASH COURSE MHT-CET CHEMISTRY

4. PRACTICE SET: Atomic Structure

\section{LEVEL-1}


TOPIC 1: Discovery of Subatomic Particles, Atomic and Isoelectronic Species


1. The number of neutrons in dipositive zinc ion with mass number 70 is

(1) 34

(2) 36

(3) 38

(4) 40


2. Which of the following pairs are isosters?

(1) \(\mathrm{CO}_{2}\) and \(\mathrm{N}_{2} \mathrm{O}\)

(2) \(\mathrm{CaO}\) and \(\mathrm{KF}\)

(3) \(\mathrm{OF}_{2}\) and \(\mathrm{HClO}\)

(4) All of these


3. What is the optimum conditions required to study the conduction of electricity through gases?

(1) High pressure and low voltage

(2) High pressure and high voltage

(3) Low pressure and high voltage

(4) Low pressure and low voltage


4. In discharge tube experiment stream of negatively charged particles travel from travel

(1) anode to cathode

(2) cathode to anode

(3) Both (1) and (2)

(4) Electrons does not


5. Which one of the following sets of ions represents the collection of isoelectronic species? \(\mathrm{Al}^{3+}, \mathrm{Cl}^{-}\)

(1) \(\mathrm{K}^{+}, \mathrm{Cl}^{-}, \mathrm{Mg}^{2+}, \mathrm{Sc}^{3+}(2) \mathrm{Na}^{+}, \mathrm{Ca}^{2+}, \mathrm{Sc}^{3+}, \mathrm{F}^{-}(3) \mathrm{K}^{+}, \mathrm{Ca}^{2+}, \mathrm{Sc}^{3+}, \mathrm{Cl}^{-}\)

(4) \(\mathrm{Na}^{+}, \mathrm{Mg}^{2+}\)


(Atomic numbers : \(\mathrm{F}=9, \mathrm{Cl}=17, \mathrm{Na}=11, \mathrm{Mg}=12, \mathrm{Al}=13, \mathrm{~K}=19, \mathrm{Ca}=20, \mathrm{Sc}=21\) )


6. Which is correct statement about proton?

(1) Proton is nucleus of deuterium

(2) Proton is a-particle

(3) Proton is ionized hydrogen molecule

(4) Proton is ionized hydrogen atom


7. Which of the following pairs of nucleides are isodiaphers?

(1) \({ }_{6}^{13} \mathrm{C}\) and \({ }_{8}^{16} \mathrm{O}\)

(2) \({ }_{1}^{1} \mathrm{H}\) and \({ }_{1}^{2} \mathrm{H}\)

(3) \({ }_{1}^{3} \mathrm{H}\) and \({ }_{2}^{4} \mathrm{He}\)

(4) \({ }_{25}^{55} \mathrm{Mn}\) and \({ }_{30}^{65} \mathrm{Zn}\)


8. What is the ratio of mass of an electron to the mass of a proton?

(1) \(1: 2\)

(2) \(1: 1\)

(3) \(1: 1837\)

(4) \(1: 3\)


9. Of the following sets which one does NOT contain isoelectronic species?

(1) \(\mathrm{BO}_{3}^{3-}, \mathrm{CO}_{3}^{2-}, \mathrm{NO}_{3}^{-}\)

(2) \(\mathrm{SO}_{3}^{2-}, \mathrm{CO}_{3}^{2-}, \mathrm{NO}_{3}^{-}\)

(3) \(\mathrm{CN}^{-}, \mathrm{N}_{2}, \mathrm{C}_{2}^{2-}\)

(4) \(\mathrm{PO}_{4}^{3-}, \mathrm{SO}_{4}^{2-}, \mathrm{ClO}_{4}^{-}\)


10. In which of the following the amount of deviation from their path in the presence of electric and magnetic field will be maximum?

(1) \(\mathrm{N}^{2-}\)

(2) \(\mathrm{N}^{3-}\)

(3) \(\mathrm{N}^{-}\)

(4) \(\mathrm{N}\)


TOPIC 2: Atomic Models, Emission and Absorption Spectrum


11. The energy of an electron in the \(n^{\text {th }}\) Bohr's orbit of hydrogen atom is

(1) \(-\frac{13.6}{n^{4}} \mathrm{eV}\)

(2) \(-\frac{13.6}{n^{3}} \mathrm{eV}\)

(3) \(-\frac{13.6}{n^{2}} \mathrm{eV}\)

(4) \(-\frac{13.6}{n} \mathrm{eV}\)


12. Which of the following does not contain number of neutrons equal to that of \({ }_{18}^{40} \mathrm{Ar}\) ?

(1) \({ }_{19}^{41} \mathrm{~K}\)

(2) \({ }_{21}^{43} \mathrm{Sc}\)

(3) \({ }_{21}^{40} \mathrm{Sc}\)

(4) \({ }_{20}^{42} \mathrm{Ca}\)


13. When an electron of charge e and mass m moves with a velocity \(\mathrm{v}\) towards the nuclear charge \(\mathrm{Ze}\) in circular orbit of radius \(r\), the potential energy of the electrons is given by

(1) \(\mathrm{Ze}^{2} / \mathrm{r}\)

(2) \(-\mathrm{Ze}^{2} / \mathrm{r}\)

(3) \(\mathrm{Ze}^{2} / \mathrm{r}\)

(4) \(\mathrm{mv}^{2} / \mathrm{r}\)


14. The Bohr's orbit radius for the hydrogen atom \((n=1)\) is approximately \(0.530 \AA\). The radius for the first excited state \((n=2)\) orbit is (in \(\AA\) )

(1) 0.13

(2) 1.06

(3) 4.77

(4) 2.12


15. According to Bohr's theory, the angular momentum of an electron in 5th orbit is

(1) \(10 \mathrm{~h} / \pi\)

(2) \(2.5 \mathrm{~h} / \pi\)

(3) \(25 \mathrm{~h} / \pi\)

(4) \(1.0 \mathrm{~h} / \pi\)


16. An electron from one Bohr stationary orbit can go to next higher orbit


(1) by emission of electromagnetic radiation.


(2) by absorption of any electromagnetic radiation. (3) by absorption of electromagnetic radiation of particular frequency.


(4) without emission or absorption of electromagnetic radiation.


17. The potential energy of electron present in ground state of \(\mathrm{Li}^{2+}\) ion is represented by:

(1) \(\frac{+3 e^{2}}{4 \pi \varepsilon_{0} r}\)

(2) \(\frac{-3 e}{4 \pi \varepsilon_{0} r}\)

(3) \(\frac{-3 e^{2}}{4 \pi \varepsilon_{0} r^{2}}\)

(4) \(\frac{-3 e^{2}}{4 \pi \varepsilon_{0} r}\)


18. The angular speed of the electron in nth orbit of Bohr hydrogen atom is

(1) directly proportional to \(\mathrm{n}\)

(2) inversely proportional of \(\sqrt{\mathrm{n}}\)

(3) inversely proportional to \(\mathrm{n}^{2}\)

(4) inversely proportional to \(\mathrm{n}^{3}\)


19. The radius of 1st Bohr's orbit for hydrogen atom is ' \(r\) '. The radius of second Bohr's orbit is

(1) \(4 \mathrm{r}\)

\((2) r^{3}\)

(3) \(4 r^{2}\)

(4) \(r^{1 / 3}\)


20. Which of the following pairs will have same chemical properties?

(1) \({ }_{6}^{14} \mathrm{C}\) and \({ }_{7}^{15} \mathrm{~N}\)

(2) \(\mathrm{O}^{2-}\) and \(\mathrm{F}^{-}\)

(3) \({ }_{18}^{40} \mathrm{Ar}\) and \({ }_{19}^{40} \mathrm{~K}\)

(4) \({ }_{17}^{35} \mathrm{Cl}\) and \(_{17}^{37} \mathrm{Cl}\)


21. If \(m\) and e are the mass and charge of the revolving electron in the orbit of radius \(r\) for hydrogen atom, the total energy of the revolving electron will be:

(1) \(\frac{1}{2} \frac{\mathrm{e}^{2}}{\mathrm{r}}\)

(2) \(-\frac{\mathrm{e}^{2}}{\mathrm{r}}\)

(3) \(\frac{m e^{2}}{\mathrm{r}}\)

(4) \(-\frac{1}{2} \frac{e^{2}}{\mathrm{r}}\)


22. Energy of an electron in a one-electron system can be calculated as


\(\mathrm{E}_{\mathrm{n}}=\frac{-2.18 \times 10^{-18} \mathrm{Z}^{2}}{\mathrm{n}^{2}}\)


Which of the following correctly states the relationship between the \(n=2\) level of He+ atom \((Z\) \(=2\) ) and \(n=2\) level of \(\mathrm{Li}^{2+}\) ion \((Z=3)\) ?

(1) \(\mathrm{E}_{\mathrm{He}^{+}}=\frac{9}{4} \mathrm{E}_{\mathrm{Li}^{2+}}\)

(2) \(\mathrm{E}_{\mathrm{He}^{+}}=\frac{4}{9} \mathrm{E}_{\mathrm{Li}^{2+}}\)

(3) \(\mathrm{E}_{\mathrm{He}^{+}}=\frac{9}{2} \mathrm{E}_{\mathrm{Li}^{2+}}\)

(4) \(\mathrm{E}_{\mathrm{He}^{+}}=\frac{2}{9} \mathrm{E}_{\mathrm{Li}^{2+}}\)


23. What is the difference between two species if one has atomic mass \(=14\) and atomic number \(=\) 7 whereas the other has atomic mass \(=14\) and atomic number \(=6\) ?

(1) Neutrons

(2) Protons

(3) Electrons

(4) All of these


24. The velocity of an electron in excited state of \(\mathrm{H}\)-atom is \(1.093 \times 10^{6} \mathrm{~m} / \mathrm{s}\). What is the circumference of this orbit?

(1) \(3.32 \times 10^{-10} \mathrm{~m}\)

(2) \(6.64 \times 10^{-10} \mathrm{~m}\)

(3) \(13.30 \times 10^{-10} \mathrm{~m}\)

(4) \(13.28 \times 10^{-8} \mathrm{~m}\)


25. Monochromatic radiation of specific wavelength is incident on \(\mathrm{H}\)-atoms in ground state. \(\mathrm{H}-\) atoms absorb energy and emit subsequently radiations of six different wavelength. Find wavelength of incident radiations:

(1) \(9.75 \mathrm{~nm}\)

(2) \(50 \mathrm{~nm}\)

(3) \(85.8 \mathrm{~nm}\)

(4) \(97.25 \mathrm{~nm}\)


26. If I excitation energy for the \(\mathrm{H}\)-like (hypothetical) sample is \(24 \mathrm{eV}\), then binding energy in III excited state is:

(1) \(2 \mathrm{eV}\)

(2) \(3 \mathrm{eV}\)

(3) \(4 \mathrm{eV}\)

(4) \(5 \mathrm{eV}\)


27. The energy of separation of an electron in a hydrogen like atom in excited state is \(3.4 \mathrm{eV}\). The de-Broglie wave length (in \(\AA\) ) associated with the electron is: (Given radius of first orbit of \(\mathrm{H}\) atom is \(0.53 \AA)\)

(1) 3.33

(2) 6.66

(3) 13.31

(4) None of these


28. If an electron undergoes transition from \(\mathrm{n}=2\) to \(\mathrm{n}=1\) in \(\mathrm{Li}^{2+}\) ion, the energy of photon radiated will be best given by

(1) \(\mathrm{h} v\)

(2) h \(v_{1}+\mathrm{h} v_{2}\)

(3) \(h v_{1}+h v_{2}+h v_{3}\)

(4) All of these 29. The photoelectric current decreases if

(1) the intensity of the source of light is decreased.

(2) the frequency of incident radiation decreases below threshold frequency.

(3) the exposure time decreases.

(4) none of these.


30. What will be the difference between electromagnetic radiation shown in A and B respectively?


(A)

(i) Velocity

(ii) Wavelength

(1) (ii) only

(2) (ii) and (iv)

(iii) Frequency

(iv) Energy

(3) (ii), (iii) and (iv)

(4) (iv) only


31. The work function of a photoelectric material is \(3.3 \mathrm{eV}\), its threshold frequency will be

(1) \(8 \times 10^{14} \mathrm{~Hz}\)

(2) \(5 \times 10^{33} \mathrm{~Hz}\)

(3) \(8 \times 10^{10} \mathrm{~Hz}\)

(4) \(4 \times 10^{11} \mathrm{~Hz}\)


32. Arrange the electromagnetic radiations a, b, c, \(d\) and e in increasing order of energy.


Frequencies of a, b and c are \(10^{15}, 10^{14}\) and \(10^{17}\) respectively whereas wavelength of (d) and (e) are \(350 \mathrm{~nm}\) and \(100 \mathrm{~nm}\) respectively?

(1) a, b, c, d, e

(2) a, b, d, e, c

(3) a, d, b, e, c

(4) b, d, a, e, c


33. For any \(\mathrm{H}\) like system, the ratio of velocities of electron in I, II and III orbit i.e., \(\mathrm{v}_{1}: \mathrm{v}_{2}: \mathrm{v}_{3}\) will be :

(1) \(1: 2: 3\)

(2) \(1: 1 / 2: 1 / 3\)

(3) \(3: 2: 1\)

(4) \(1: 1: 1\)


34. In which of the following Bohr's stationary state, the electron will be at maximum distance from the nucleus?

(1) IInd

(2) Ist

(3) Vth

(4) IIIrd


35. What is the potential energy of an electron present in \(\mathrm{N}\)-shell of the Be ion?

(1) \(-3.4 \mathrm{eV}\)

(2) \(-6.8 \mathrm{eV}\)

(3) \(-13.6 \mathrm{eV}\)

(4) \(-27.2 \mathrm{eV}\)


36. Which of the following transitions of electrons in the hydrogen atom will emit maximum energy?

(1) \(n_{5} \rightarrow n_{4}\)

(2) \(n_{4} \rightarrow n_{3}\)

(3) \(n_{3} \rightarrow n_{2}\)

(4) all will emit same


energy


37. What is the ratio of time periods \(\left(\mathrm{T}_{1} / \mathrm{T}_{2}\right)\) in second orbit of hydrogen atom to third orbit of \(\mathrm{He}^{+}\)ion?

(1) \(8 / 27\)

(2) \(32 / 27\)

(3) \(27 / 32\)

(4) None of these


38. Bohr's model is not able to account for which of the following.

(1) Stability of atom.

(2) Spectrum of neutral helium atom.

(3) Energy of free electron at rest.

(4) Calculation of radii of the stationary states.


TOPIC 3: Planck's Quantum Theory, Heisenberg's Uncertainty Principle and Schrodinger's Wave Equation:


39. If the energy of a photon is \(3.03 \times 10^{-19} \mathrm{~J}\) then, the wavelength \((\lambda)\) of the photon is :

(1) \(6.56 \mathrm{~nm}\)

(2) \(65.6 \mathrm{~nm}\)

(3) \(656 \mathrm{~nm}\)

(4) \(0.656 \mathrm{~nm}\)


40. The value of Planck's constant is \(6.63 \times 10^{-34} \mathrm{Js}\). The velocity of light is \(3.0 \times 10^{8} \mathrm{~ms}^{-1}\). Which value is closest to the wavelength in nanometers of a quantum of light with frequency of \(8 \times\) \(10^{15} \mathrm{~s}^{-1}\)



(1) \(3 \times 10^{7}\)

(2) \(2 \times 10^{-25}\)

(3) \(5 \times 10^{-18}\)

(4) \(4 \times 10^{1}\)


41. The de Broglie wavelength of a tennis ball of mass \(60 \mathrm{~g}\) moving with a velocity of 10 metres per second is approximately Planck's constant, \(\mathrm{h}=6.63 \times 10^{-34} \mathrm{Js}\)

(1) \(10^{-31}\) metres

(2) \(10^{-16}\) metres

(3) \(10^{-25}\) metres

(4) \(10^{-33}\) metres


42. Wavelength associated with electron motion

(1) increases with increase in speed of electron.

(2) remains same irrespective of speed of electron.

(3) decreases with increase of speed of \(\mathrm{e}^{-}\)(electron).

(4) is zero.


43. The Heisenberg uncertainty principle will be most significant for which of the following object?

(1) Object A of mass \(9.11 \times 10^{-30} \mathrm{~kg}\)

(2) Object B of mass \(9.11 \times 10^{-28} \mathrm{~g}\)

(3) Object C of mass \(9.11 \times 10^{-24} \mathrm{mg}\)

(4) Object D of mass \(9.11 \times 10^{-28} \mathrm{~kg}\)


44. The momentum (in \(\mathrm{kg}-\mathrm{m} / \mathrm{s}\) ) of photon having \(6 \mathrm{MeV}\) energy is :

(1) \(3.2 \times 10^{-21}\)

(2) 2.0

(3) \(1.6 \times 10^{-21}\)

(4) None of these


45. When electronic transition occurs from higher energy state to lower energy state with energy difference equal to \(\Delta \mathrm{E}\) electron volts, the wavelength of the line emitted is approximately equal to

(1) \(\frac{12395}{\Delta \mathrm{E}} \times 10^{-10} \mathrm{~m}\)

(2) \(\frac{12395}{\Delta \mathrm{E}} \times 10^{10} \mathrm{~m}\)

(3) \(\frac{12395}{\Delta \mathrm{E}} \times 10^{-10} \mathrm{~cm}\)

(4) \(\frac{12395}{\Delta \mathrm{E}} \times 10^{10} \mathrm{~cm}\)


46. Which of the following statement concerning probability density \(\left(\psi^{2}\right)\) and radial distribution function


\(\left(4 \pi r^{2} \psi^{2}\right)\) for a s-orbital of H-like species is correct?


(1) \(\psi^{2}\) is minimum at nucleus but \(4 \pi r^{2} \psi^{2}\) is maximum at nucleus.


(2) \(\psi^{2}\) is maximum at nucleus but \(4 \pi r^{2} \psi^{2}\) is minimum at nucleus.


(3) Both \(\psi^{2}\) and \(4 \pi r^{2} \psi^{2}\) are maximum at nucleus. (4) Both \(\psi^{2}\) and \(4 \pi r^{2} \psi^{2}\) are minimum at nucleus.


47. The angular momentum of \(d\) electron is

(1) \(\frac{h}{2 \pi} \sqrt{6}\)

(2) \(\frac{h}{\pi} \sqrt{6}\)

(3) \(\frac{\mathrm{h}}{2 \pi} \sqrt{2}\)

(4) \(\frac{h}{\pi} \sqrt{2}\)


48. If \(E_{1}, E_{2}\), and \(E_{3}\) represent respectively the kinetic energies of an electron and an alpha particle and a proton each having same de-Broglie wavelength then

(1) \(\mathrm{E}_{1}>\mathrm{E}_{3}>\mathrm{E}_{2}\)

(2) \(\mathrm{E}_{2}>\mathrm{E}_{3}>\mathrm{E}_{1}\)

(3) \(\mathrm{E}_{1}>\mathrm{E}_{2}>\mathrm{E}_{3}\)

(4) \(\mathrm{E}_{1}=\mathrm{E}_{2}=\mathrm{E}_{3}\)


49. If uncertainty in position and momentum are equal, then uncertainty in velocity is :

(1) \(\frac{1}{2 m} \sqrt{\frac{h}{\pi}}\)

(2) \(\sqrt{\frac{\mathrm{h}}{2 \pi}}\)

(3) \(\frac{1}{\mathrm{~m}} \sqrt{\frac{\mathrm{h}}{\pi}}\)

(4) \(\sqrt{\frac{h}{\pi}}\)


50. Which of the following statement is wrong about photon ?

(1) Photon's energy is \(h v\).

(3) Momentum of photon is \(\frac{h v}{\mathrm{c}}\)

(4) Photon exerts no pressure.


(2) Photon's rest mass is zero.


51. Excited hydrogen atom emits light in the ultraviolet region at \(2.47 \times 10^{15} \mathrm{~Hz}\). With this frequency, the energy of a single photon is: \(\left(\mathrm{h}=6.63 \times 10^{-34} \mathrm{Js}\right)\)

(1) \(8.041 \times 10^{-40} \mathrm{~J}\)

(2) \(2.680 \times 10^{-19} \mathrm{~J}\)

(3) \(1.640 \times 10^{-18} \mathrm{~J}\)

(4) \(6.111 \times 10^{-17} \mathrm{~J}\)


52. Ionization energy of gaseous \(\mathrm{Na}\) atoms is \(495.5 \mathrm{~kJ} \mathrm{~mol}^{-1}\). The lowest possible frequency of light that

ionizes a sodium atom is \(\left(\mathrm{h}=6.626 \times 10^{-34} \mathrm{Js}, \mathrm{N}_{\mathrm{A}}=6.022 \times 10^{23} \mathrm{~mol}^{-1}\right)\)

(1) \(7.50 \times 10^{4} \mathrm{~s}^{-1}\)

(2) \(4.76 \times 10^{14} \mathrm{~s}^{-1}\)

(3) \(3.15 \times 10^{15} \mathrm{~s}^{-1}\)

(4) \(1.24 \times 10^{15} \mathrm{~s}^{-1}\) TOPIC 4: Quantum Numbers, Electronic Configuration and Shape of Orbitals


53. Which one of the following set of quantum numbers is not possible for \(4 \mathrm{p}\) electron?

(1) \(\mathrm{n}=4, \ell=1, \mathrm{~m}=-1, \mathrm{~m}_{\mathrm{s}}=+\frac{1}{2}\)

(2) \(\mathrm{n}=4, \ell=1, \mathrm{~m}=0, \mathrm{~m}_{\mathrm{s}}=+\frac{1}{2}\)

(3) \(\mathrm{n}=4, \ell=1, \mathrm{~m}=2, \mathrm{~m}_{\mathrm{s}}=+\frac{1}{2}\)

(4) \(\mathrm{n}=4, \ell=1, \mathrm{~m}=-1, \mathrm{~m}_{\mathrm{s}}=-\frac{1}{2}\)


54. What is the correct orbital designation of an electron with the quantum number, \(n=4, l=3, m\) \(=-2\) \(s=1 / 2 ?\)

(1) \(3 s\)

(2) \(4 f\)

(3) \(5 p\)

(4) \(6 s\)


55. Which orbital of the following is lower in energy in a many electron atom?

(1) \(2 s\)

(2) \(3 d\)

(3) \(4 s\)

(4) \(5 f\)


56. Which of the following graph correspond to one node


  1. e
(3)


57. The total number of electrons that can be accommodated in all orbitals having principal quantum number 2 and azimuthal quantum number 1 is

(1) 2

(2) 4

(3) 6

(4) 8


58. What can be the representation of the orbital having 3 angular nodes and \(n=5\).

(1) \(5 d\)

(2) \(5 f\)

(3) \(5 p\)

(4) \(5 \mathrm{~s}\)


59. The five \(d\)-orbitals are designated as \(d_{x y}, d_{y z}, d_{x z}, d_{x^{2}-y^{2}}\) and \(d_{z^{2}}\). Choose the correct statement.


(1) The shapes of the first three orbitals are similar but that of the fourth and fifth orbitals are different.


(2) The shapes of all five d-orbitals are similar.


(3) The shapes of the first four orbitals are similar but that of the fifth orbital is different.


(4) The shapes of all five d-orbitals are different.


60. Maximum number of electrons in a subshell of an atom is determined by the following:

(1) \(2 l+1\)

(2) \(4 l-2\)

(3) \(2 n^{2}\)

(4) \(4 l+2\)


61. An e- has magnetic quantum number as -3 , what is its principal quantum number?

(1) 1

(2) 2

(3) 3

(4) 4


62. For \(a, f\)-orbital, the values of \(m\) are

(1) \(-2,-1,0,+1,+2\)

(2) \(-3,-2,-1,0,+1,+2,+3\)

(3) \(-1,0,+1\)

(4) \(0,+1,+2,+3\)


63. A \(5 f\) orbital has

(1) one node

(2) two nodes

(3) three nodes

(4) four nodes.


64. If electron has spin quantum number \(+1 / 2\) and a magnetic quantum number -1 , it cannot be present in

(1) \(d\)-orbital

(2) f-orbital

(3) \(p\)-orbital

(4) s-orbital.


65. The orbital angular momentum for an electron revolving in an orbit is given by \(\sqrt{l(l+1)} \cdot \frac{h}{2 \pi}\). This momentum for an s-electron will be given by

(1) zero

(2) \(\frac{h}{2 \pi}\)

(3) \(\sqrt{2} \cdot \frac{h}{2 \pi}\)

(4) \(+\frac{1}{2} \cdot \frac{h}{2 \pi}\)


66. The energy of the electron in \(\mathrm{Be}^{3+}\) ion depends on 




(1) the principal quantum number only.


(2) the principal and azimuthal quantum numbers only.


(3) the principal, azimuthal and magnetic quantum numbers only.


(4) the principal, azimuthal, magnetic and spin quantum numbers.


67. What are the component values (in terms of \(h / 2 \pi\) ) of the orbital angular momentum along the Z-direction for a \(2 p\) electron?

(1) \(+\frac{1}{2},-\frac{1}{2}\)

(2) \(+\frac{3}{2},+\frac{1}{2},-\frac{1}{2},-\frac{3}{2}\)

(3) \(+2,+1,0,-1,-2\)

\((4)+1,0,-1\)


68. The total number of orbitals associated with the principal quantum number 5 is :

(1) 20

(2) 25

(3) 10

(4) 5


69. The total spin and magnetic moment for the atom with atomic number 24 are:

(1) \(\pm 3, \sqrt{48} \mathrm{BM}\)

(2) \(\pm 3, \sqrt{35} \mathrm{BM}\)

(3) \(\pm \frac{3}{2}, \sqrt{48} \mathrm{BM}\)

(4) \(\pm \frac{3}{2}, \sqrt{35} \mathrm{BM}\)


70. A principal shell having the highest energy subshell to be ' \(g\) ' can accomodate electrons to a maximum of

(1) 18

(2) 32

(3) 25

(4) 50